A mysterious factorial equation.

  Рет қаралды 68,332

Michael Penn

Michael Penn

Күн бұрын

We solve a nice equation over the natural numbers involving exponentiation and factorials. Our approach employs Wilson's theorem and other modular reduction tricks.
Please Subscribe: kzbin.info...
Merch: teespring.com/stores/michael-...
Personal Website: www.michael-penn.net
Randolph College Math: www.randolphcollege.edu/mathem...
Randolph College Math and Science on Facebook: / randolph.science
Research Gate profile: www.researchgate.net/profile/...
Google Scholar profile: scholar.google.com/citations?...
If you are going to use an ad-blocker, considering using brave and tipping me BAT!
brave.com/sdp793
Buy textbooks here and help me out: amzn.to/31Bj9ye
Buy an amazon gift card and help me out: amzn.to/2PComAf
Books I like:
Abstract Algebra:
Judson(online): abstract.ups.edu/
Judson(print): amzn.to/2Xg92wD
Dummit and Foote: amzn.to/2zYOrok
Gallian: amzn.to/2zg4YEo
Artin: amzn.to/2LQ8l7C
Differential Forms:
Bachman: amzn.to/2z9wljH
Number Theory:
Crisman(online): math.gordon.edu/ntic/
Strayer: amzn.to/3bXwLah
Andrews: amzn.to/2zWlOZ0
Analysis:
Abbot: amzn.to/3cwYtuF
How to think about Analysis: amzn.to/2AIhwVm
Calculus:
OpenStax(online): openstax.org/subjects/math
OpenStax Vol 1: amzn.to/2zlreN8
OpenStax Vol 2: amzn.to/2TtwoxH
OpenStax Vol 3: amzn.to/3bPJ3Bn
My Filming Equipment:
Camera: amzn.to/3kx2JzE
Lense: amzn.to/2PFxPXA
Audio Recorder: amzn.to/2XLzkaZ
Microphones: amzn.to/3fJED0T
Lights: amzn.to/2XHxRT0
White Chalk: amzn.to/3ipu3Oh
Color Chalk: amzn.to/2XL6eIJ

Пікірлер: 105
@prime_suntereh2241
@prime_suntereh2241 3 жыл бұрын
Wilson + inequality + bionomial + basic congruence = amazing problem
@HarmonicEpsilonDelta
@HarmonicEpsilonDelta 3 жыл бұрын
10:47 I think we used the hypothesis when we multiplied 2*(n/2), because for that to be true, we need both 2 and (n/2) in the product, and for that, we need n/2 to be different than 2, since they come from the factorial, and therefore n>4, which implies n\geq 5.
@goodplacetostop2973
@goodplacetostop2973 3 жыл бұрын
12:03 14:23
@caesar_cipher
@caesar_cipher 3 жыл бұрын
Something very important happened at 12:03 - u must check it out
@goodplacetostop2973
@goodplacetostop2973 3 жыл бұрын
Caesar Cipher Indeed! Thanks 👍
@R0M4ur0
@R0M4ur0 3 жыл бұрын
In general, in this video we have a "that's a good place to stop", quantity squared
@goodplacetostart9099
@goodplacetostart9099 3 жыл бұрын
Good place to start at 0:00
@CM63_France
@CM63_France 3 жыл бұрын
@@R0M4ur0 along with "let's go head and do that","all the way {up|down} to","and so on and so forth"....
@factsverse9957
@factsverse9957 3 жыл бұрын
It appeared on the SMO (Singapore Mathematical Olympiad), Open Round 2 on the year 2008, as problem number 1. Hope it helps!
@factsverse9957
@factsverse9957 3 жыл бұрын
@Adam Romanov You can find the pdf of it online, I got a pdf from a website fadjarp3g, plug in SMO Round 2 2008 fadjarp3g in the Google search bar and I think you should find it, hope this helps
@yusrizalakbar3983
@yusrizalakbar3983 3 жыл бұрын
14:00 he said "that's true for n≥5" but 4^4 > 4!, why is it I'm sorry if it's a dumb questions
@yusrizalakbar3983
@yusrizalakbar3983 3 жыл бұрын
@qu-ack oh okay thanks
@roboto12345
@roboto12345 3 жыл бұрын
I was trying that exact problem, it was in an LTE article. Thanks
@chaitanyagaur7928
@chaitanyagaur7928 3 жыл бұрын
Training for maths olympiad?
@swayamprakashkar9664
@swayamprakashkar9664 3 жыл бұрын
Very nice solution... Nailed it👏👏
@Kettwiesel25
@Kettwiesel25 2 жыл бұрын
I wasn't aware of Wilson's Theorem, but it's not too hard to see that n+1 has to be a prime in this problem, this is the easy part of Wilson's Theorem. If n+1 has a factor, which is smaller than it, then it appears in n! as well as (n+1)^k, so the two can not be just 1 apart (they are not coprime)
@ffggddss
@ffggddss 3 жыл бұрын
First thing I thought when I saw the thumbnail: "Hey, that looks like Wilson's Theorem! . . Oh wait, not quite." Wilson's Theorem says that p | [(p-1)! + 1], iff p is prime. Here we would make n = p-1 and this becomes (n+1) | (n! + 1), iff (n+1) is prime; vs our problem: (n+1)ᵏ = n! + 1 which is a stronger condition on (n! + 1) than mere divisibility by (n+1). k & n are restricted to the +ve integers, so start with k=1: N+1 - 1 - n = n! Works for n = 1 and 2; no others. Solutions: (n,k) = (1,1), (2,1) k=2: n! + 1 will have to be a square. I think the only cases of that are n = 4, 5, and 7. n = 4 works for this equation; 5 and 7 do not: 4! + 1 = 25 = 5² 5! + 1 = 121 = 11² 7! + 1 = 5041 = 71² So: (4+1)² - 1 = 4! . . 24 = 24 Solution: (n,k) = (4,2) I suspect there are no others, but I don't see a way to prove that. Let's see what Michael has... ... Very nicely done! So my suspicion was borne out. Thanks to Michael for the proof. Fred
@mihamihailovic4519
@mihamihailovic4519 3 жыл бұрын
If it isn't too much of a hastle can you please put the video link in the description for theorems you used and proved in another video (like wilsons theorem here)
@a_llama
@a_llama 3 жыл бұрын
10:47 im guessing n>5 because you assume that there exists an n/2 >= 3, meaning n>=6
@UndeadFil
@UndeadFil 3 жыл бұрын
Correct, and since n+1 has to be prime, the case n=5 can be ruled out because 6 obviously isn't prime.
@mariomestre7490
@mariomestre7490 2 жыл бұрын
Una equació meravellosa. Genial
@angelvalera1997
@angelvalera1997 3 жыл бұрын
I love those videos. Greetings from Spain :D
@mathissupereasy
@mathissupereasy 3 жыл бұрын
I would love to see you working on inequality problems.
@newkid9807
@newkid9807 3 жыл бұрын
Le Family Patriarch Bao Quoc Le
@TwilightBrawl59
@TwilightBrawl59 3 жыл бұрын
3:25 Not a big mistake but the left hand side is even and the right hand side is odd.
@kuldeepparashar7266
@kuldeepparashar7266 3 жыл бұрын
Thanks sir
@jamesfortune243
@jamesfortune243 3 жыл бұрын
Seeing your proofs is making me smarter. :)
@mcwulf25
@mcwulf25 2 жыл бұрын
Clever solution. Especially noticing n/2 and 2 are factors of n! when n even.
@pendawarrior
@pendawarrior 3 жыл бұрын
Fantastic
@TheFlash-bw3hi
@TheFlash-bw3hi 3 жыл бұрын
Woow dope explanation.... Can u please try Putnam 2014 a5 , the official solution is unbearable 😭
@math_infinity_
@math_infinity_ 3 жыл бұрын
Good question
@TechToppers
@TechToppers 3 жыл бұрын
That was brain storming! Uff Will see again...
@thayanithirk1784
@thayanithirk1784 3 жыл бұрын
Please do Putnam 2006 B2
@prunodagen
@prunodagen 3 жыл бұрын
You don"t need Wilson Theorem. (n+1)^k = n! + 1 n! + 1 is odd for n > 1, then n + 1 must be odd, then n must be even. (we don't need n + 1 prime, just the fact n is even to get k = 0[n])
@Miju001
@Miju001 3 жыл бұрын
Woo, that's slick
@spiderjerusalem4009
@spiderjerusalem4009 3 ай бұрын
"just the fact that n is even to get k=0" how?
@TechToppers
@TechToppers 3 жыл бұрын
Let s(n) denote the sum of the digits of a positive integer n in base 10. If s(m) = 20 and s(33m) = 120, what is the value of s(3m)? Can someone please help me with this? It came in India. PRMO 2019 25th August Paper
@WriteRightMathNation
@WriteRightMathNation 3 жыл бұрын
Nice problem, but at 3:28 (someone else may have already pointed this out...), you mixed up "left-hand" and "right-hand"... 😊
@abhishekbhattacharjee4039
@abhishekbhattacharjee4039 3 жыл бұрын
This problem is beautiful
@joaopedrobmenezes2977
@joaopedrobmenezes2977 3 жыл бұрын
Vp(x!) >= 2*Vp(x) for all non prime x such that x> 4, then you can see that if n>4 and it is not prime( which is the case for n >4) k would have to be at least n, but (n + 1)^n - 1 > n! for n > 4. This is just an alternative solution:)
@gregsavitt7176
@gregsavitt7176 3 жыл бұрын
Just got out of Calc 3 thinking that I had gotten pretty good at math. Then I watched this and my brain melted.
@MysticMagik
@MysticMagik 3 жыл бұрын
This is mere child's play compared with calc 3
@spiderjerusalem4009
@spiderjerusalem4009 3 ай бұрын
​@@MysticMagikquite the opposite. Lots of calc stuffs are straight-up chug-in-plug-in dumb play, whilst number theory and combinatorics are whole different unpredictable beast
@chhabisarkar9057
@chhabisarkar9057 3 жыл бұрын
A nice little problem from INMO 2014 , problem 2 Prove that [n] + [n/2] + [n/3] ....+ [n/n] + [√n] is even , where [n] is the floor of n Plz see it . Thank you
@Kettwiesel25
@Kettwiesel25 2 жыл бұрын
Proof sketch: Notice that f(n) - f(n-1), the increase from n-1 to n, is the number of factors of n, except if n is a square number, then it is 1 more. Now use that a number has an odd number of factors if and only if it is a square.
@TechToppers
@TechToppers 2 жыл бұрын
Hello 😂
@chhabisarkar9057
@chhabisarkar9057 2 жыл бұрын
@@TechToppers hmmmm
@spiderjerusalem4009
@spiderjerusalem4009 3 ай бұрын
Let f(n) be the above expression f(n-1) =Σⁿ⁻¹ₖ₌₁ ⌊ⁿ⁻¹/ₖ⌋ + ⌊√(n-1)⌋ = Σⁿ⁻¹ₖ₌₁ (⌈ⁿ/ₖ⌉-1) + ⌊√(n-1)⌋ (Here, we used the fact that ⌈ⁿ/ₖ⌉=⌊ⁿ⁻¹/ₖ⌋+1 for any n,k∈ℕ) f(n)-f(n-1) = n-Σⁿₖ₌₁ (⌈ⁿ/ₖ⌉-⌊ⁿ/ₖ⌋) +⌊√n⌋-⌊√(n-1)⌋ Let Δ(k,n)=1 if k|n, 0 otherwise hence ⌈ⁿ/ₖ⌉-⌊ⁿ/ₖ⌋=1-Δ(k,n) summing from k=1 to n, it equals Σⁿₖ₌₁ (1-Δ(k,n)) = n-Σₖₗₙ 1 = n-τ(n) Thus f(n)-f(n-1)=τ(n)+⌊√n⌋-⌊√(n-1)⌋ •》Case 1: n is perfect square implying ⌊√n⌋=1+⌊√(n-1)⌋ hence f(n)-f(n-1)=τ(n)+1, which is even since τ(n) is odd if and only if n is a perfect square •》n is not perfect square implying ⌊√n⌋=⌊√(n-1)⌋ hence f(n)-f(n-1)=τ(n), which ia even in either case, f(n)-f(n-1)=k for some even k summing from 2 to n f(n)=(n-1)k+f(1) = (n-1)k+2 = 2((n-1)(k/2)+1) which is even for any n∈ℕ, as desired.
@stevenwilson5556
@stevenwilson5556 3 жыл бұрын
6:53 why is n + 1 a prime? Does this cover cases where n + 1 is not prime?
@chhabisarkar9057
@chhabisarkar9057 3 жыл бұрын
Reverse wilson
@subversively6680
@subversively6680 3 жыл бұрын
By applying Wilson 's theorem, can we find the rest of prime numbers that are >100 (theoretically speaking)
@ruairigarrett6834
@ruairigarrett6834 3 жыл бұрын
Good luck calculating those factorials lol
@iabervon
@iabervon 3 жыл бұрын
@@ruairigarrett6834 You only need them mod p+1, so at least you're not dealing with numbers more than about p², but you're still doing p multiplications and might as well just try dividing p by everything less than p.
@hans-juergenbrasch3683
@hans-juergenbrasch3683 3 жыл бұрын
n=1,2 imply k=1. Now for n>2 we must have n+1=p is a prime, otherwise n+1 would have a proper divisor 1 < d =< (n+1)/2 < n which yields a contradiction modulo d: 0=n!=(n+1)^k-1=-1 and hence we can rewrite p^k-1=(p-1)! with p>=5 or 1+p+...+p^(k-1)=(p-2)! Note that gcd(1+p+...+p^(k-1),p-1)=gcd(k,p-1) Since (p-1)/2 < p-2 we must have k = t (p-1)/2 for some t. Since p^(p-1)-1= = (1+(p-1))^(p-1)-1>(p-1)^(p-1) we must have t=1, i.e. p^((p-1)/2)-1=(p-1)! For p=5 we get a solution corresponding to n=4, k=2 and for p>5 we get p^((p-1)/2)-1= =prod_(i=1)^((p-1)/2) [i(p-i)]> >[p^2+(p-3)^2-5]p^[(p-1)/2)-2]> >p^((p-1)/2), a contradiction. Note, that we have split the product into two factors, one for i=1,2 given by (p-1)2 (p-2) and the other for i=3,..., (p-1)/2
@user-A168
@user-A168 3 жыл бұрын
Good
@sylviabayard6117
@sylviabayard6117 3 жыл бұрын
A way to skip Wilson's Theorem: Add 1 to both sides, to get (n+1)^k = n! + 1 Well, we know that n! + 1 is coprime to any number less than or equal to n, so any primes dividing (n+1)^k must be greater than or equal to n+1. However, by the very form of it, any primes dividing (n+1)^k must be less than or equal to n+1. Therefore the ONLY prime dividing (n+1)^k must be n+1.
@factsverse9957
@factsverse9957 3 жыл бұрын
Then? How can we proceed, it's unclear for me
@blackmephistopheles2273
@blackmephistopheles2273 3 жыл бұрын
@@factsverse9957 Continue the argument in the video, after the point where Wilson's Theorem was used; this argument above shows that WT is unnecessary. So, continue from 6:56 (edited to find time reference).
@Pedro-fg4sw
@Pedro-fg4sw 3 жыл бұрын
LTE Lemma
@spiderjerusalem4009
@spiderjerusalem4009 3 ай бұрын
but... that is just a rephrase of the converse of wilson's theorem which was applied in the vid....
@rustemtehmezov9494
@rustemtehmezov9494 3 жыл бұрын
Where is guy says everytime "answer=1"?
@masonholcombe3327
@masonholcombe3327 3 жыл бұрын
Video idea: Prove the Hardy-Ramanajan Series for partitions of n :)
@geometrydashmega238
@geometrydashmega238 3 жыл бұрын
He has a playlist of partitions and I think he was going to do that, or did it already. Check that out
@jideofororamah4399
@jideofororamah4399 3 жыл бұрын
Because K is the power and n=2 (n+1)^k = n!. 1 is a constant so how did you get k=1 where n=2
@RealLifeKyurem
@RealLifeKyurem 3 жыл бұрын
3^k = 3 What k satisfies this equation?
@minh9545
@minh9545 3 жыл бұрын
@@RealLifeKyurem what do the 3 stripes and the -1(mod p) mean?
@RealLifeKyurem
@RealLifeKyurem 3 жыл бұрын
TNT ≡ is read “is congruent to.” -1 mod p means if you divide a number n by a prime p, the remainder is -1. In other words, it satisfies the expression: apm - 1; for any integer a. The reason why ≡ is used instead of =, is because any multiple of p satisfies the expression.
@LiangLao2
@LiangLao2 3 жыл бұрын
just a suggestion, can you open a new channel? one channel focuses on math question especially the Olympic question, one channel focus math theory such as analysis algebra number theory, etc.
@Polpaccio
@Polpaccio 2 жыл бұрын
1 year later , 200k more subs
@donaldbiden7927
@donaldbiden7927 3 жыл бұрын
Beautiful solution ! Kindly tell the motivation for the inequality.
@ericslavich4297
@ericslavich4297 3 жыл бұрын
Once we know k is a multiple of n, you look at the original equation again. You realize the exponent on the left hand side is at least n. Also note n^n > n!, so the left hand side is growing more rapidly than the right. The rest is details and being rigorous.
@accountname1047
@accountname1047 3 жыл бұрын
The form of the numbers is similar to the brown numbers
@xinfuchen5355
@xinfuchen5355 3 жыл бұрын
No Wilsons Theorem is needed. When n>4, i) (n+1)^k= 1+n!= 1 mod (2). This imples that n is even ii) n even implies that (n-1)! = 0 mod (n) iii) k = ((n+1)^k-1)/n mod(n) = (n-1)! mod (n) = 0 mod(n) (iva) k=0, no (ivb) k>=n, Then (n+1)^k > n^n -1> n!-1. No solution, Hence, when n>4, there is no solution.
@Teodyk
@Teodyk 2 жыл бұрын
Obviously.
@rustemtehmezov9494
@rustemtehmezov9494 3 жыл бұрын
You get (n+1)^mn-1>nⁿ>n! but isn't it was (n+1)^mn-1=n! and this means n!>nⁿ? İ think it was typo
@oussemabouaneni992
@oussemabouaneni992 3 жыл бұрын
That's the contradiction. He supposed that such an n exists and proved that its existence is impossible.
@rustemtehmezov9494
@rustemtehmezov9494 3 жыл бұрын
@@oussemabouaneni992 but if we get n!>nⁿ then this will be contradiction, because this never holds for these rules. But he got nⁿ>n! which is clearly true. He should be write n!>nⁿ (he got it at the end but wrote it wrongly i think, but i understood his proof method clear)
@oussemabouaneni992
@oussemabouaneni992 3 жыл бұрын
@@rustemtehmezov9494 No no he wrote it perfectly correctly. He proved that (n+1)^k - 1 > n^n > n! which means that (n+1)^k - 1 is never equal to n! for n >= 5 which is what he set out to prove.
@rustemtehmezov9494
@rustemtehmezov9494 3 жыл бұрын
@@oussemabouaneni992 Oh, now i understood what you mean, yeah this is accetable. (Almost same as i supposed) Thanks for clarification👍
@oussemabouaneni992
@oussemabouaneni992 3 жыл бұрын
@@rustemtehmezov9494 Cheers mate.
@tomholroyd7519
@tomholroyd7519 3 жыл бұрын
Count Olaf
@user-vk6nl6mi1d
@user-vk6nl6mi1d 2 жыл бұрын
0 not a natural number?
@mcbeaulieu
@mcbeaulieu 3 жыл бұрын
What is Mod? 😭😭😭😭
@kruksog
@kruksog 3 жыл бұрын
Mod gives remainder. Some examples: 5 mod 3 = 2 because 5/3 has remainder 2. 4 mod 2 = 0 because 4 is divisible by 2 (i.e. no remainder). Once again, 7 mod 2 = 1 because 7/2 has a remainder of 1. It's important to remember when doing elementary number theory, we work in the integers and naturals. Hence all the whole number concerns.
@devrajdas6544
@devrajdas6544 3 жыл бұрын
Is there any problem you can't solve?
@vaxjoaberg9452
@vaxjoaberg9452 3 жыл бұрын
Global climate change?
@xinfuchen5355
@xinfuchen5355 3 жыл бұрын
No Wilson's Theorem is needed. When n>4, i) (n+1)^k= 1+n!= 1 mod (2). This implies that n is even ii) n even and n>4 imply that (n-1)! = 0 mod (n) iii) k = ((n+1)^k-1)/n mod(n) = (n-1)! mod (n) = 0 mod(n) (iva) k=0, no (ivb) k>=n, Then (n+1)^k-1 >=( n+1)^n -1>=n^n>n!. No solution, Hence, when n>4, there is no solution.
@seroujghazarian6343
@seroujghazarian6343 3 жыл бұрын
For integers, n^n=>n! Is true. But beyond that, it's not always true. Case in point:(1/2)^(1/2)=sqrt(2)/2 < (1/2)! = sqrt(pi)/2
@HagenvonEitzen
@HagenvonEitzen 3 жыл бұрын
13:33 Or just (n+1)^n -1 >= (n+1)n(n-1)... 2 - =(n+1)! -1 > n!
@mandelbrotsugee
@mandelbrotsugee 3 жыл бұрын
じっと見ていると、n=4と分かりますな。 K=2
@tyjensen8366
@tyjensen8366 3 жыл бұрын
God he’s such a chad
@guibaroleo
@guibaroleo 3 жыл бұрын
hahahahahaha
@sdspivey
@sdspivey 3 жыл бұрын
You say K must be a multiple of N, but the solutions given have N as a multiple of K.
@axelperezmachado3500
@axelperezmachado3500 3 жыл бұрын
all solutions with n >= 5 must have k as a multiple of n. That does not necessarily apply to the given solutions.
@jideofororamah4399
@jideofororamah4399 3 жыл бұрын
I got confused at how you got k=1 at n=2
@ddp8511
@ddp8511 3 жыл бұрын
Just put n=2 U get 3^k=3
@jideofororamah4399
@jideofororamah4399 3 жыл бұрын
Angel Mendez-Rivera That solves it. Thanks 🙏❤️ @michael penn
@keithmasumoto9698
@keithmasumoto9698 3 жыл бұрын
Immediately I don't get why n+1 is prime. If n=5 then n+1=6 is not prime.
@RandomGamer9
@RandomGamer9 3 жыл бұрын
I had to rewatch to understand it but if you reduce (n+1)^k-1=n! (mod n+1) you get 0-1=n! (mod n+1) then you rearrange to get the form from Wilson's theorem and that n+1 is prime. The zero in the above equation come from the fact that (n+1)^k is divisible by (n+1)
@CM63_France
@CM63_France 3 жыл бұрын
Hi, This was before hair cutting, so I understand my request about the camera is not taken into account.
The smallest such prime...
16:44
Michael Penn
Рет қаралды 56 М.
Too hard for the IMO? Too easy?
24:20
Michael Penn
Рет қаралды 96 М.
WORLD'S SHORTEST WOMAN
00:58
Stokes Twins
Рет қаралды 46 МЛН
Я обещал подарить ему самокат!
01:00
Vlad Samokatchik
Рет қаралды 6 МЛН
Iron Chin ✅ Isaih made this look too easy
00:13
Power Slap
Рет қаралды 36 МЛН
A number theory problem from Morocco!
20:08
Michael Penn
Рет қаралды 65 М.
What is the factorial of -½?
12:46
Stand-up Maths
Рет қаралды 568 М.
Researchers thought this was a bug (Borwein integrals)
17:26
3Blue1Brown
Рет қаралды 3,4 МЛН
The Reason Train Design Changed After 1948
13:05
Joe Scott
Рет қаралды 187 М.
Swedish Mathematics Olympiad | 2002 Question 4
14:19
Michael Penn
Рет қаралды 310 М.
Nature's Incredible ROTATING MOTOR (It’s Electric!) - Smarter Every Day 300
29:37
The moment we stopped understanding AI [AlexNet]
17:38
Welch Labs
Рет қаралды 807 М.
New Zealand Mathematical Olympiad 2019 Question 5
18:42
Michael Penn
Рет қаралды 64 М.
FUN Ecuadorian Math Olympiad Number Theory Problem
19:50
Michael Penn
Рет қаралды 42 М.
WORLD'S SHORTEST WOMAN
00:58
Stokes Twins
Рет қаралды 46 МЛН